ANSWER SHEET

Practice Test 2

1. images

2. images

3. images

4. images

5. images

6. images

7. images

8. images

9. images

10. images

11. images

12. images

13. images

14. images

15. images

16. images

17. images

18. images

19. images

20. images

21. images

22. images

23. images

24. images

25. images

26. images

27. images

28. images

29. images

30. images

31. images

32. images

33. images

34. images

35. images

36. images

37. images

38. images

39. images

40. images

41. images

42. images

43. images

44. images

45. images

46. images

47. images

48. images

49. images

50. images

51. images

52. images

53. images

54. images

55. images

 

Practice Test 2

SECTION 1: MULTIPLE-CHOICE QUESTIONS

TIME 80 MINUTES

55 QUESTIONS

Please note that the order of the types of multiple-choice questions may be spread out on the actual test.

Section 1 is Worth 50 Percent of the Test

 

Directions: Each of the following questions has four choices. Choose the best response and record your answer on the answer sheet on page 245. There will be no penalty for incorrect answers in the multiple-choice section. Answer as many multiple-choice questions in the time permitted, even if you are unsure of the correct answer.

 

Questions 1–2 refer to the following graph.

Percentage of Frequent Use of Media by Candidate Supporters in 2016

images

1. Which of the following conclusions is accurate based on the information presented in the bar chart?

(A) Clinton supporters use new media more than Sanders supporters.

(B) Trump supporters accuse cable media outlets of promoting “fake news.”

(C) Cable was a major factor in the political socialization of Trump voters.

(D) The national media had a bias against Trump and Cruz during the 2016 campaign.

2. Which of the following trends is reflected in the bar graph?

(A) The Republican candidates relied more on the national media than the Democratic candidates did.

(B) Even though late-night comedians use their monologues to satirize the news, supporters of the 2016 candidates used it less frequently than other media outlets.

(C) The Internet has been used less extensively compared to political advertising by supporters of the 2016 candidates.

(D) Local media is used more by supporters of Republican candidates than supporters of Democratic candidates.

Questions 3–4 refer to the following line graph.

images

Source: U.S. Census Bureau

3. Which of the following choices reflect the information provided in the chart?

(A) Hispanic voter turnout has been greater than non-Hispanic black turnout between 1986 and 2012.

(B) There was a significant increase of non-Hispanic white voter turnout following presidential elections.

(C) Non-Hispanic black voter turnout in presidential elections showed the greatest increase of any of the highlighted ethnic groups between 1986 and 2012.

(D) Voter turnout in all groups has consistently declined in presidential elections since 2004.

4. Which of the following conclusions about voter turnout is accurate based on the information provided in the chart?

(A) Hispanic voter turnout in midterm elections has consistently risen since the 1980s.

(B) Hispanics voted more in presidential elections than midterm elections because there was an increase in undocumented Hispanics in the country.

(C) The gap between non-Hispanic white and non-Hispanic black voter turnout in presidential elections has narrowed significantly since 2000.

(D) More non-Hispanic blacks voted in the 2008 presidential election than in the 2010 midterm election because many states reduced early voting opportunities.

Questions 5–6 are based on the following pie charts.

images

5. Which of the following is an accurate statement about the pie charts?

(A) The percentage of white senators is higher than the percentage of male House members.

(B) There is a higher percentage of black House members than female senators.

(C) There is a higher percentage of female House members than female senators.

(D) There is a higher percentage of Asian House members than Hispanic senators.

6. Which of the following best explains why there is a discrepancy between the percentage of male and female House and Senate members?

(A) Fewer females are qualified to run for the Senate and House.

(B) The electorate has not demanded more female representation in Congress.

(C) Female officeholders do not run for the House and Senate because they do not get leadership positions after they are elected.

(D) Male senate candidates who are elected have less political experience than female senate candidates.

Questions 7–8 refer to the following graph.

images

Source: Congressional Budget Office

7. Which of the following is reflected by the information in the graph?

(A) The percentage of Medicaid spending has increased more than the percentage of Medicare and Social Security spending.

(B) The percentage of Social Security spending has been higher than the percentage of Medicare and Medicaid spending and is projected to remain so.

(C) The rise and percentage of Social Security spending are outpacing the rise in Medicare spending.

(D) The greatest percentage jump in total federal entitlement spending as part of the Gross Domestic Product is projected to occur between 2020 and 2030.

8. Which of the following is a consequence of federal entitlement spending increasing between 2005 and 2050?

(A) The average life expectancy of senior citizens will get lower.

(B) Medicare and Medicaid will get funded by the states.

(C) There will be new funding for discretionary programs by Congress.

(D) Congress must pass new legislation to guarantee the solvency of entitlement programs.

Questions 9–10 refer to the following table.

Approval Ratings and Reelection Rates of House and Senate

Congress

Reelection Rates

Year

Approval

House

Senate

2000

24%

98%

79%

2001

26%

 

 

2002

29%

96%

86%

2003

29%

 

 

2004

30%

98%

96%

2005

22%

 

 

2006

19%

94%

79%

2007

14%

 

 

2008

12%

94%

83%

2009

17%

 

 

2010

11%

85%

84%

2011

12%

 

 

2012

13%

90%

91%

2013

10%

 

 

2014

7%

95%

82%

2015

17%

 

 

2016

16%

97%

87%

2017

19%

 

 

9. Based on the information presented in the table, what should be the reelection rate trend of House and Senate incumbents starting in 2018?

A. House incumbents should be reelected in a smaller percentage than Senate incumbents.

B. Most House and Senate incumbents will lose their reelection bids.

C. House incumbents and Senate incumbents should be reelected at the same percentage.

D. House incumbents should be reelected in a greater percentage than Senate incumbents.

10. Based on the information presented in the table, what conclusion can you reach about approval ratings and reelection rates of House and Senate incumbents?

(A) Even though public opinion polls show that Congress is unpopular, people still vote for their sitting representatives and senators.

(B) Reelection rates of House and Senate incumbents are significantly lower when Congress has a lower approval rating.

(C) Reelection rates of House and Senate incumbents are higher during presidential election years even if there are low approval ratings of Congress.

(D) The approval rating of incumbent House members and Senators is usually similar to the public approval rating of Congress.

Questions 11–14 are based on the following passage.

Section 1.

“All persons born or naturalized in the United States, and subject to the jurisdiction thereof, are citizens of the United States and of the state wherein they reside. No state shall make or enforce any law which shall abridge the privileges or immunities of citizens of the United States; nor shall any state deprive any person of life, liberty, or property, without due process of law; nor deny to any person within its jurisdiction the equal protection of the laws.”

—Section 1 The Fourteenth Amendment to the United States Constitution

11. Which part of the Fourteenth Amendment guarantees that the treatment of citizens in one state must be the same as a person in another state?

(A) Persons born or naturalized in the United States are citizens.

(B) States cannot make laws that violate the privileges or immunities of citizens in the United States.

(C) States cannot deny any person the equal protection of the law.

(D) States cannot deny people in their state of life, liberty, or property without due process of law.

12. Which constitutional principle was established through the due process clause of the Fourteenth Amendment?

(A) Popular sovereignty

(B) Federalism

(C) Eminent domain

(D) Selective incorporation

13. For which of the following court cases did a majority of Supreme Court justices rely on the equal protection clause of the Fourteenth Amendment in making their decision?

(A) United States v Lopez (1995)

(B) Roe v Wade (1973)

(C) Baker v Carr (1961)

(D) Shaw v Reno (1961)

14. Based upon your knowledge of the Fourteenth Amendment, which of the following statements is accurate?

(A) The Fourteenth Amendment was adopted in response to segregation laws that existed in the South after the Civil War.

(B) The Fourteenth Amendment’s equal protection clause can only be used to protect individuals against violations of federal laws.

(C) Children of undocumented immigrants born in the United States are not citizens.

(D) The due process clause of the Fourteenth Amendment can only be applied if the due process clause of the Fifth Amendment is violated.

Questions 15–17 refer to the following passage.

Emily’s List

“Our vision is a government that reflects the people it serves, and decision makers who genuinely and enthusiastically fight for greater opportunity and better lives for the Americans they represent. We will work for larger leadership roles for pro-choice Democratic women in our legislative bodies and executive seats so that our families can benefit from the open-minded, productive contributions that women have consistently made in office…. “Every day. Every election victory. We are helping to put women into office who have the power to put progressive change to work,” EMILY’s List’s vision is to be a driving force of change in America. By electing more pro-choice Democratic women to national, state and local office, EMILY’s List will consistently infuse our government with leaders who will drive change. Change that truly matters today, tomorrow and forever.”

—Ellen R. Malcolm, Founder and Chair of Emily’s List

15. Which of the following arguments reflects the group’s mission?

(A) To attract pro-choice Democratic and Republican women candidates to run for office

(B) To support progressive change from pro-choice Democratic women elected to office

(C) To join other progressive special-interest groups to defeat Republican candidates running for office

(D) To sue corporations that are accused of paying men more than women doing the same job

16. Which of the following is an example of “putting progressive change to work”?

(A) Supporting the right to own guns for the purpose of protecting your family

(B) Supporting a law that would allow the development of clean coal

(C) Supporting the concept of home schooling

(D) Supporting the Violence Against Women Act

17. Which of the following groups would be most closely aligned with Emily’s List?

(A) The U.S. Chamber of Commerce

(B) National Abortion Reproductive Rights Action League (NARAL)

(C) Americans United for Life

(D) Family Research Council

Questions 18–19 refer to the following map.

images

18. The map shows those states that have voter identification requirements. Which of the following explains why the Democratic Party has challenged voter identification laws?

(A) Voter identification being required to vote will lead to fewer cases of voter fraud.

(B) Voter identification being required to vote will suppress minority turnout.

(C) Voter identification being required to vote will increase voter turnout.

(D) Voter identification being required to vote will increase the college vote.

19. Which of the following is a valid argument in favor of the passage of voter ID laws?

(A) There would be no need for states to provide provisional ballots.

(B) Voter rights need to be protected.

(C) Voter identification policies would increase voter turnout.

(D) Voter identification policies can be used if a state has early voting.

Questions 20–21 refer to the following cartoon.

images

20. Which of the following represents the point of view of this political cartoon?

(A) The reapportioned legislative districts must be ruled unconstitutional.

(B) There are more Democratic gerrymandered districts than Republican.

(C) Gerrymandering as a political practice resulted in the unfair drawing of legislative districts.

(D) Gerrymandering is a legitimate political tool carried out by the state legislature.

21. Which of the following Supreme Court cases dealt with the issue raised by the cartoon?

(A) Shaw v Reno (1993)

(B) Citizens United v Federal Election Commission (2010)

(C) Gideon v Wainwright (1963)

(D) Marbury v Madison (1803)

Questions 22–23 refer to the following table.

Amount of Money Spent in Presidential and Congressional Elections 1998–2016

Cycle

Total Cost of Election

Congressional Races

Presidential Race

2016*

$6,917,636,161

$4,266,514,050

$2,651,122,110

2014

$3,845,393,700

$3,845,393,700

N/A

2012

$6,285,557,223

$3,664,141,430

$2,621,415,792

2010

$3,631,712,836

$3,631,712,836

N/A

2008

$5,285,680,883

$2,485,952,737

$2,799,728,146

2006

$2,852,658,140

$2,852,658,140

N/A

2004

$4,147,304,003

$2,237,073,141

$1,910,230,862

2002

$2,181,682,066

$2,181,682,066

N/A

2000

$3,082,340,937

$1,669,224,553

$1,413,116,384

1998

$1,618,936,265

$1,618,936,265

N/A

Source: Federal Election Commission

22. Based on the information in the chart, what would be a major factor in the increase of the total cost of elections from 1998 to 2016?

(A) Increased public financing of presidential and congressional elections

(B) Laws passed that increased the amount of soft money candidates could raise

(C) Supreme Court decisions that equate money donated to candidates as a First Amendment right

(D) An increase in political contributions to candidates by foreign countries

23. Based on the information in the chart, which of the following actions would a candidate do to raise the most money to run a winning campaign?

(A) Create a Political Action Committee (PAC)

(B) Participate in candidate town meetings

(C) Run political ads asking for contributions

(D) Hire political consultants

24. The Voting Rights Act of 1965 directed the Justice Department to

(A) implement a literacy test for all eligible voters in the South.

(B) monitor state voting where there had been instances of discrimination.

(C) redraw voting districts where there was no minority representation.

(D) take control of voter registration in the South.

25. Which of the following is a result of media-centered coverage of political campaigns?

(A) A greater loyalty to political parties by the electorate

(B) Voter satisfaction with the candidates running for office

(C) Less of a reliance by the voters on social media

(D) A decrease in the analysis of policy issues by the media

26. Which of the following best illustrates the implied power of Congress?

(A) Passing the budget for the Defense Department

(B) Passing the Civil Rights Act of 1964

(C) Confirming John Roberts as Chief Justice of the Supreme Court

(D) Overriding a presidential veto

27. When a potential candidate is deciding whether to run for president, there are many factors to consider before the official announcement is made. Which of the following steps would the candidate take first before deciding to run?

(A) Buying political ads in his or her home state

(B) Picking a running mate

(C) Creating a campaign slogan

(D) Book speaking engagements in early caucus and primary states

28. The president wants a clean air, clean water law that places restrictions on the amount of carbon emissions that can be produced by factories. Which of the following steps would the president take to get support for passage of the law?

(A) Pressure states to pass their own laws prohibiting the production of goods that cause carbon emissions

(B) Direct the Environmental Protection Agency to develop regulations that would reduce carbon emissions

(C) Ask the attorney general to investigate factories that are producing excessive amounts of carbon emissions

(D) Make a proposal to Congress during the State of the Union Address

29. A Senator is against a proposed piece of legislation that would mandate background checks for gun purchases at gun shows. Recent polls show that 90 percent of the public favors the law, and after a recent mass shooting it appears as if Congress may pass the law. What can the Senator do to stop the bill from coming to a vote?

(A) Lobby the Supreme Court to rule that the bill would be unconstitutional

(B) Filibuster the proposed law

(C) Call for a cloture vote

(D) Have other senators sign a discharge petition

30. Which of the following statements about the Electoral College is accurate?

(A) If no presidential candidate receives a majority of the electoral votes, the House of Representatives determines who will be the president.

(B) If a presidential candidate wins the popular vote but the other candidate wins the majority of the Electoral College, the House and Senate will determine the winner.

(C) If there is a tie in the Electoral College, the Senate determines the winner.

(D) Electors must vote for the presidential and vice-presidential candidates who received the most votes in their state.

31. The fact that there is a free exercise clause in the First Amendment of the Constitution illustrates that

(A) free speech conflicts with the separation of church and state.

(B) wearing religious clothing is prohibited in public schools.

(C) faith-based initiatives are unconstitutional.

(D) there is a balance between secular and religious interests.

32. In deciding the landmark Supreme Court case Brown v Board of Education (1954), the court overturned the separate but equal doctrine precedent established in Plessy v Ferguson (1896). Which of the following describes the philosophy used by the justices in deciding the Brown case?

(A) Constitutional originalism

(B) Judicial Activism

(C) Judicial Restraint

(D) Judicial oversight

33. Which of the following explains how bureaucracies operate to implement public policy?

(A) Bureaucracies have the ability to abolish a poorly functioning department in another agency.

(B) Bureaucracies have the ability of determining and spending their own budgets.

(C) Bureaucracies have the ability of coordinating their own resources and personnel to achieve policy goals.

(D) Bureaucracies provide oversight to Congress before it passes laws affecting their agency.

34. Which of the following situations could be a violation of the privacy clause of the Fourth Amendment based on prior Supreme Court decisions?

(A) The police obtain a search warrant to search a cell phone to find out whom a suspect called.

(B) A citizen is put into solitary confinement for getting into a fight while serving time in prison.

(C) A state passes a law that bans abortions before the twentieth week of a pregnancy.

(D) A college bans housing that allows males and females to live in the same room.

35. Which of the following actions taken by an individual or group would violate the Supreme Court ruling Engel v Vitale (1962)?

(A) A Bible study after-school group is funded by the school district.

(B) A clergyman is invited to say a prayer at graduation at a public high school.

(C) A comparative religion course is taught in a public school.

(D) Amish parents not wanting to send their children to public school after age 16.

36. The No Child Left Behind Act of 2001 and Every Student Succeeds Act of 2015 established educational standards that states must follow if they want federal funding for local educational programs. Many local school boards objected to these laws. Which of the following constitutional provisions did local school officials believe the federal government violated and overstepped its authority?

(A) The Privilege and Immunity clause of the Fourteenth Amendment

(B) The Tenth Amendment’s Reserved Power clause

(C) The Full Faith and Credit clause in Article IV

(D) The Supremacy Clause in Article VI

37. The landmark case Marbury v Madison (1803) established the principle of judicial review after the Supreme Court ruled a congressional law unconstitutional as a result of the fact that the case was brought directly to the court. Which of the following situations represents the best example of a case dealing with original jurisdiction?

(A) A review by the Supreme Court over a dispute between New York and New Jersey dealing with property rights related to Ellis Island

(B) A review of the constitutionality of the Violence Against Women’s Act

(C) A review of President Trump’s executive order banning immigrants from certain predominantly Muslim countries

(D) A review of a federal law mandating affirmative action in industries that have contracts with the government

38. A president has many positions to fill in the executive branch of government. Which of the following is an example of a position that follows the constitutional doctrine of advise and consent?

(A) Appointing the White House press secretary

(B) Appointing the White House chief of staff

(C) Appointing the White House counsel to the president

(D) Appointing the ambassador to Japan

39. A presidential daily tracking poll has candidate A ahead in the swing state of Ohio by 3 points with a margin of error of plus or minus 3.5 points. Exit polls have the same candidate behind by 3 points.

Which of the following statements represents the difference between a tracking poll and exit polls?

(A) The tracking poll has too large a margin of error.

(B) The tracking poll does not use a scientific sample.

(C) Exit polls have a much larger sample.

(D) Exit polls rely on a sample of actual votes in swing districts.

40. A defense corporation lobbyist wants funding in the defense budget that would result in a large government contract for that corporation. Which of the following statements represents how the iron-triangle network would accomplish this goal?

(A) The lobbyist would testify before the Senate Foreign Relations Committee, lobby influential Congressmen, and get the support from the Defense Department.

(B) The lobbyist would meet with the White House Chief of Staff, take a member from the Senate Foreign Relations Committee to an expensive dinner, and pay for a political ad that supports his or her position.

(C) The lobbyist would seek the aid of state officials, testify before the Senate Foreign Relations Committee, and meet with a high-ranking officer from the Defense Department.

(D) The lobbyist would submit a position paper to the Washington Post, testify before the Senate Foreign Relations Committee, and meet with the White House chief of staff.

41. President Jones is serving a second term. A month after he/she gives a final State of the Union speech, the opposition party is calling the president a lame duck. Which of the following would be the most difficult for the president to achieve in the final year?

(A) Getting Congress to pass emergency funding for hurricane victims

(B) Issuing an executive order to give federal employees paid family leave

(C) Getting the Senate to approve a Supreme Court nominee

(D) Getting Congress to raise the debt ceiling

42. One of the core beliefs about the United States government is that citizens should have equality of opportunity. Which of the following statements reflects that belief?

(A) It is the individual’s responsibility to provide for his or her family without government assistance.

(B) The government should guarantee access to health care.

(C) The government should guarantee that everybody has a college education.

(D) The government should guarantee that everybody has a job.

43. Which of the following is an accurate comparison of the Anti-Federalist paper Brutus 1 and Federalist No. 10?

 

Brutus 1

Federalist No. 10

(A)

Supported “necessary and proper clause”

Feared the influence of factions

(B)

Supported the Supremacy clause

Against the Supremacy clause

(C)

Supported inclusion in the Constitution of a Bill of Rights

Feared majority would threaten minority rights

(D)

Favored Congress having the right to tax

Against Congress having the right to tax

44. Which of the following is an accurate comparison of Baker v Carr (1961) and Shaw v Reno (1993)?

 

Baker v Carr (1961)

Shaw v Reno (1993)

(A)

Ruled that rural voters had to be represented proportionally to urban voters

Ruled that gerrymandered districts were legal

(B)

Established the “one person, one vote” doctrine

Declared majority-minority districts unconstitutional

(C)

Allowed state legislatures to gerrymander districts

Ruled that Voting Rights Act was constitutional

(D)

Violated separation of powers doctrine between state legislatures and the courts

Ruled that race-based voting was no different than race-based hiring

45. Which of the following is an accurate comparison of a delegated power of the president and an enumerated power of Congress

 

Delegated Power of the President

Enumerated Power of Congress

(A)

Power to declare war

Power to regulate interstate commerce

(B)

Power to grant pardons

Power to appoint justices

(C)

Power to issue executive orders

Power to make all laws necessary and proper

(D)

Power to carry out duties as Commander in Chief

Power to establish rules of naturalization

46. Which of the following is an accurate comparison of the philosophies of judicial activism and judicial restraint?

 

Judicial Activism

Judicial Restraint

(A)

Supreme Court overturns stare decisis

Supreme Court upholds precedent

(B)

Supreme Court accepts limited number cases for certiorari

Supreme Court hears more than 200 cases for oral arguments

(C)

Supreme Court allows amicus curie briefs

Supreme Court limits amicus briefs

(D)

Supreme Court allows live TV coverage of oral arguments

Supreme Court allows taping of oral arguments

47. Which of the following is an accurate comparison of Republican Party and Democratic Party philosophy regarding the environment?

 

Republican Party

Democratic Party

(A)

The federal government should allow clean coal production

The federal government should pass laws regulating carbon emissions

(B)

The United States should be part of climate change treaties.

The United States should abandon climate control treaties.

(C)

The Environmental Protection Agency should set strict standards on clean air and clean water

The Environmental Protection Agency should reduce clean air and water regulations

(D)

The federal government should support an oil pipeline between Canada and the United States

The federal government should open up national parks for private investment

48. Which of the following is the main difference between the Iowa caucus and most primaries?

(A) A primary is closed to non-party members, while anybody can vote in a caucus.

(B) More states use the caucus system rather than primaries.

(C) Most primaries are held before the Iowa caucus.

(D) Speeches are made for candidates at caucus sites before there is actual voting.

49. Which of the following provisions of the Voting Rights Act of 1965 directed the Justice Department to take action?

(A) Implement a literacy test for all eligible voters in the South

(B) Monitor state voting where there had been instances of discrimination

(C) Redraw voting districts where there was no minority representation

(D) Take control of voter registration in the South

50. What usually happens when one party controls the executive branch of government and another party controls one or both houses of the legislative branch?

(A) The president has expanded power to get proposals through Congress.

(B) The White House executive departments issue more policies.

(C) Compromise must take place for legislation to be signed into law.

(D) Congress becomes the stronger branch of government.

51. Which of the following scenarios occurs when earmarks are attached to a bill?

(A) The bill reduces the deficit.

(B) The bill gives a congressperson the right to cut off debate.

(C) A congressperson adds a project to an existing bill for the purpose of helping his or her district.

(D) The bill is sent back to committee for further amendments.

52. Which of the following are guaranteed by the Fifth Amendment and when a policeman reads a suspect’s Miranda rights?

(A) Directs the police to inform the suspect that he or she has due-process rights

(B) Prohibits the police from obtaining evidence without a search warrant

(C) Guarantees the right to an attorney if the accused cannot afford one

(D) Guarantees the accused the right to challenge any witnesses

53. If a person advocates devolution, which of the following constitutional provisions would support legislation to accomplish that goal?

(A) The Reserved Powers clause of the Tenth Amendment

(B) The Full Faith and Credit clause in Article 4

(C) The Equal Protection clause of the Fourteenth Amendment

(D) The Eminent Domain clause in the Fifth Amendment

54. What happens as a result of the increase in the number of social media outlets?

(A) The public relies more on the mainstream media.

(B) The reliability of the news is lessened.

(C) There is a greater concentration of news media ownership.

(D) The Internet is regulated by the government to ensure accuracy.

55. Which of the following occurs when a congressional committee provides oversight?

(A) The committee reviews the budget and actions of a bureaucratic agency.

(B) The committee is charged with drafting an appropriations bill.

(C) The committee sets a time limit for debate of a bill that places restrictions on a bureaucratic agency.

(D) The committee debates whether there is sufficient evidence to impeach a president.

SECTION 2: FREE-RESPONSE QUESTIONS

TIME: 100 MINUTES

Section 2 is Worth 50 Percent of the Test

 

Directions: Answer all four of the following questions in 100 minutes. Questions 1–3 should be written in 20 minutes. Question 4 should be written in 40 minutes. The questions are based on your knowledge of U.S. government and politics, and questions contain scenarios, materials from charts, graphs, and tables you will have to analyze and draw conclusions from. Question 4 is an argumentation question that requires you to take a position. Make sure you give specific and sufficient information in your answers. Please number them clearly on your answer sheets.

 

Question 1 (20 Minutes)

The U.S. Census Bureau delivered its planned subjects for the 2020 Census to Congress, which include gender, age, race, ethnicity, relationship, and homeownership status.

By law, the Census Bureau must deliver decennial census subjects to Congress three years before Census Day, with the next one occurring April 1, 2020. The subjects represent the necessary balance between the need for data, and the Census Bureau’s commitment to reduce the time it takes to complete the form. By law, the actual questions that will appear on the 2020 Census questionnaire must be submitted to Congress by March 31, 2018.

“Our goal is a complete and accurate census,” Census Bureau Director John H. Thompson said. “In planning for the 2020 Census, the Census Bureau has focused on improving its address list by using imagery, finding ways to increase household self-response, leveraging resources inside and outside the government, and making it easier and more efficient for census takers to complete their work. Furthermore, for the first time ever, the decennial will offer an online response option with the ultimate goal of improving question design and data quality while addressing community concerns.”

—Press Release from the United States Census Bureau, March 28, 2017

After reading the passage, answer Parts A, B, and C.

A. Describe where the Census Bureau gets its authority to address the issues raised in the passage.

B. Based on your answer in Part A, explain how the Census Bureau is planning on executing that authority and describe one problem that could arise as a result of the way the census is going to be taken.

C. In the context of this passage, explain what the consequences of the 2020 census will have on the political institutions of the state and federal governments.

Question 2 (20 minutes)

This question asks you to compare a required Supreme Court case with another Supreme Court case that is summarized below, and provides the necessary information that you need to answer the question.

The State of Pennsylvania passed the Pennsylvania Abortion Control Act of 1988–1989. The law mandated that a woman seeking an abortion had to give “informed consent” at least 24 hours prior to the procedure warning her that an abortion could be detrimental to her health. The woman seeking abortion had to notify her husband about the procedure, and if the woman was a minor, she would need parental consent or seek judicial bypass instead of consent. The law also had a medical emergency definition that provided for an abortion for the woman if the pregnancy threatened her life or created a serious health risk.

In the case that followed this law, Planned Parenthood of Southeastern Pennsylvania v Casey (1992), the court was split 5–4 on the constitutionality of Roe v Wade (1973) and the constitutionality of the law itself based upon the standards set fourth in Roe v Wade. The court upheld the principles established by Roe applying the doctrine of stare decisis to the case. The court also ruled that the state had a legitimate right to make laws regarding abortion as long as they did not create an “undue burden” on the woman’s right to have an abortion. Thus, the court upheld the provisions of the Pennsylvania law with the exception of spousal notification.

A. Identify the constitutional clause used by the Supreme Court in Planned Parenthood v Casey (1992) and Roe v Wade (1973).

B. Based on the constitutional issues identified in (A), explain in which ways the facts and decision of Roe v Wade (1973) was different from the decision reached in Planned Parenthood v Casey.

C. Describe the judicial consequences of the decision reached in Planned Parenthood v Casey and give an example of how a special-interest group reacted to the decision.

images

Source: Office of Management and Budget

Use the information provided in the graph to answer the following questions.

Question 3 (20 Minutes)

A. Identify the most common feature of the federal budget between 1982 and 2012.

B. Describe the deviation found in the U.S. federal government income and outlay between 1998 and 2000 and explain why this trend happened.

C. Explain how this graphic as shown illustrates the principles of United States fiscal policy from 2002 to 2012.

Question 4 (40 minutes)

The Declaration of Independence and the Constitution of the United States outline how a limited government is best achieved.

Describe an argument that explains which of the ideas that contributed to the concept of a limited government—natural rights, popular sovereignty, and the social contract—best accomplishes the goals set forth in the Declaration of Independence and the United States Constitution.

In your essay, you must do the following:

Make a valid argument or thesis that addresses the question and presents a sound basis of reasoning.

Support your argument with at least TWO examples of accurate and relevant information. One piece of evidence from one of the following foundational documents:

—Federalist No. 10

—the Declaration of Independence

—the U.S. Constitution

The second piece of evidence must be from another foundational document from the list above or from your study of how the government carries out its functions.

Use logical arguments to explain why your proof supports your argument/thesis.

Respond to an opposing point of view or alternative argument refuting, conceding, or rebutting that point of view.

ANSWER KEY

Practice Test 2

1. C

2. B

3. C

4. C

5. A

6. B

7. D

8. D

9. D

10. A

11. C

12. D

13. D

14. A

15. B

16. D

17. B

18. B

19. B

20. C

21. A

22. C

23. A

24. B

25. D

26. B

27. D

28. D

29. B

30. A

31. D

32. B

33. C

34. C

35. B

36. B

37. A

38. D

39. D

40. A

41. C

42. B

43. C

44. B

45. D

46 A

47. A

48. D

49. B

50. C

51. C

52. A

53. A

54. B

55. A

 

ANSWERS EXPLAINED

Section 1: Multiple-Choice Questions

1. (C) The graph is a breakdown of what percentage of Clinton, Sanders, Cruz, and Trump supporters use different types of print, broadcast, and social media as well as relying on family and friends for political information. Political socialization is defined as how an individual develops a political point of view. The media and family and friends are components of political socialization, with family and friends playing a more influential role. The chart shows that more than 60 percent of Trump supporters frequently watched cable television. Choice (A) is incorrect because new media, the Internet, was used by a lower proportion of Clinton supporters than Sanders supporters. Choice (B) is incorrect because even though Trump supporters accuse certain cable outlets of promoting “fake news,” they still watch cable stations like the Fox News Channel, which they perceive, in large numbers, as more pro-Trump. Choice (D) is incorrect because there is no evidence from the chart that the national media had a bias against Trump or Cruz.

2. (B) Other than political advertising, which is not a media outlet but is shown on media outlets, supporters of political candidates view late-night comedians much less than national media, cable, local media, newspapers, radio, and the Internet. The late-night comedians overall have smaller viewing audiences than many daytime or prime-time news shows, and because they are viewed as more critical of Republicans as evidenced by the almost nonexistent viewership by Trump and Cruz supporters. Choices (A) and (C) are incorrect because they are inaccurate statements. There is no evidence from the chart that Choice (D) is correct, as supporters of Clinton and Sanders use local media more than supporters of Trump and Cruz.

3. (C) The chart illustrates the turnout rates of non-Hispanic white voters, non-Hispanic black voters, Hispanic voters, and other ethnic voters in non-presidential and presidential elections from 1986 to 2014. Non-Hispanic black voter turnout jumped from mid-40 percent to the near 70-percent range between 1988 and 2012, far more than the increase of other groups. Choice (A) is incorrect because non-Hispanic black voter turnout has been consistently higher than Hispanic voter turnout in those years. Choice (B) is incorrect because there has been a drop-off in Hispanic turnout following every presidential election. Choice (D) is incorrect because there has been an increase in turnout most notably among non-Hispanic black voters since 2004.

4. (C) Historically, non-Hispanic whites turned out in higher percentages than non-Hispanic blacks in presidential elections, but by 2004, that gap almost disappeared. By 2008 and 2012, when the first African-American candidate, Barack Obama, was on the presidential ballot, black voters turned out in greater percentages than non-Hispanic white voters. Choice (A) is incorrect because Hispanic turnout in midterm elections dropped to its lowest level since 1986 in 2014. Choice (B) is incorrect because undocumented Hispanics cannot vote. Choice (D) is incorrect because non-Hispanic black voters always vote in lower numbers in midterm elections compared to presidential elections.

5. (A) The chart illustrates the gender and racial composition of the 114th Congress. It is apparent from the chart that both the House and Senate have a much greater male composition than female composition, and both Houses are predominantly white. This has been the basis of criticism of Congress by women and minority groups who believe they are not represented proportionally to the population as a whole. Whites make up 94 percent of the Senate, and males make-up 80.6 percent of the House. Choices (B), (C), and (D) are all incorrect statistics that are not represented in the chart.

6. (B) Ever since women were granted the right to vote by the Nineteenth Amendment to the U.S. Constitution, voters have always elected more men to Congress than women. One reason is that there have been many more male candidates than female candidates. Another reason is that the electorate as a whole have preferred male candidates to female candidates. This has been changing since 1992, which was called the “year of the woman” because of the record number of women candidates elected. After the sexual-assault scandals in 2018, a record number of women candidates ran for political offices across the country. Choices (A) and (C) are incorrect statements. There is no evidence to support choice (D).

7. (D) The chart illustrates the amount of federal entitlement spending on Medicare, Medicaid, and Social Security as a percentage of the Gross Domestic Product, which is the total amount of goods and services produced in a year. As the amount of federal entitlement spending increased from 2005 to 2010 and is projected to increase even more from 2020 to 2050, it has become a greater percentage of the gross domestic product. Choices (A), (B), and (C) are incorrect because they do not reflect statistically the information presented in the chart.

8. (D) If Social Security, Medicare, and Medicaid are going to remain solvent and available, Congress will need to pass legislation that enables these programs to function. This legislation can take the form of budgetary reductions in those programs, means testing or raising the age for Social Security recipients, or other reforms that will slow down the growth of the programs. A factor working against the long-term solvency of the safety-net programs is the fact that more people are living longer and reaching the age where they will qualify for Social Security and Medicare or depend on Medicaid. Choice (A) is incorrect because the life expectancy of Americans is getting higher. Choice (B) is incorrect because Medicare is not funded by the states and Medicaid is only partially funded by the states. Choice (C) is incorrect because funding for discretionary programs has been decreasing.

9. (D) The chart illustrates the relationship between the approval ratings of Congress and reelection rates of House and Senate members from 1992 to 2016, both in midterm and presidential elections. Therefore, you can predict that in 2018 there should be a higher percentage of House incumbents reelected than Senate incumbents. Regardless of the type of election or the fact that the highest approval rating of Congress in 2004 was 30 percent, Senate incumbents get reelected at a rate from 79 percent to 96 percent, and House incumbents at a reelection rate that ranges from 88 percent to 98 percent. The reasons for this include gerrymandered districts favoring the reelection of incumbents, and that most people, even though they disapprove of the job that Congress is doing, still support their own Senator or House member. The reason there is a higher reelection rate of House incumbents compared to Senate incumbents is that House member districts can be gerrymandered while Senators represent the entire state. Choices (A), (B), and (C) are statistically inaccurate.

10. (A) As stated in the explanation of question nine, the low approval ratings of Congress do not result in lower reelection rates of House and Senate incumbents. Choices (B), (C), and (D) are incorrect because the answers do not reflect the relationship between low approval ratings and reelection rates of House and Senate members.

11. (C) The Fourteenth Amendment’s equal protection clause is a required foundational document. It is important to understand that there are other sections of the Fourteenth Amendment that have a profound impact on the civil liberties of individuals and the protection of the rights of citizens against the abuses of the state and federal governments. The equal protection clause guarantees that states cannot deny any person the equal protection of the laws. The result is that a person in one state must be treated the same as a person in another state when it comes to the application of law. The equal protection clause has become the bedrock of the rule of law and has advanced the cause of civil rights. Choices (A), (B), and (D) are incorrect because those are not the sections of the Fourteenth Amendment that apply to equal protection. They apply to other aspects of law, natural-born citizens, privileges and immunities, and due process.

12. (D) Selective incorporation, or what has also been called the “nationalization of the Bill of Rights,” applied the due process clause of the Fourteenth Amendment to other amendments of the Bill of Rights and applied the decision nationally. For example, when the Supreme Court decided that the police must get a valid search warrant to obtain admissible evidence in a criminal trial in the state of Ohio, that decision in Mapp v Ohio (1961) applied to the entire country. Choice (A) is incorrect because popular sovereignty relates to the principle of people having a voice in their government. Voting emerged from this principle. Choice (B) is incorrect because the principle of federalism gives states the right to make laws that are not delegated to the federal government. Choice (C) is incorrect because eminent domain is the principle of law that allows the government to take away private property if that property is to be used for the public good and the owner is paid.

13. (D) The equal protection clause of the Fourteenth Amendment has become the basis of the advancement of civil rights. In the required Shaw v Reno (1962) case, the Supreme Court ruled that gerrymandered majority-minority legislative districts violated the equal protection clause because the end result of drawing a district that had a majority of African Americans resulted in the election of an African American. Therefore, a secondary result was that all the other districts had a majority of white voters that resulted in these districts electing whites, thus violating the equal protection clause. Choice (A) is incorrect because United States v Lopez (1995) was a federalism case that dealt with Congress overstepping its authority by passing a gun-free school zone act. Choice (B) is incorrect because Roe v Wade (1973) dealt with the legalization of abortion based on the right of privacy contained in the Fourth Amendment, and choice (C) is incorrect because Baker v Carr (1961) was a case that emerged as a result of a violation of the clause in the Constitution addressing the size of legislative districts.

14. (A) The Thirteenth, Fourteenth, and Fifteenth Amendments were passed in response to the discrimination against the former slaves before the Civil War and continued after the Civil War when Southern states passed Black Codes, literacy tests and other laws that were discriminatory. The Thirteenth Amendment freed the slaves, the Fourteenth Amendment defined citizenship, gave citizens equal protection, and guaranteed citizens due process of law, and the Fifteenth Amendment gave former slaves the right to vote. Choice (B) is incorrect because the equal protection clause protects individuals from abuses by the state or federal government. Choice (C) is incorrect because any person born in the United States is a citizen. Choice (D) is incorrect because the due process clause of the Fourteenth Amendment can be used if a state law is discriminatory, and the due process clause of the Fifth Amendment can be used if a federal law is discriminatory.

15. (B) Emily’s List is a special-interest group that works to get, endorse, and campaign for female Democratic candidates running for local, state, and national office. The group supports a progressive agenda and specifically an agenda that furthers the cause of women’s rights such as pay equality and the right of a woman to choose whether she wants to have an abortion. Choice (A) is incorrect because Emily’s List supports pro-choice Democrats over any Republican running for office. Choice (C) is incorrect because there is no formal alliance with other progressive special-interest groups who are working to defeat Republicans. Choice (D) is incorrect because, even though Emily’s List supports pay equality, it is not part of the group’s stated mission.

16. (D) Besides pro-choice issues, Emily’s list also supports progressive causes such as equal pay, paid family leave, and family health care. The Violence Against Women Act of 1994 is a United States federal law signed by President Bill Clinton. The Act provided $1.6 billion toward investigation and prosecution of violent crimes against women, imposed automatic and mandatory restitution on those convicted, and allowed civil redress in cases prosecutors chose to leave unprosecuted. The Act also established the Office on Violence Against Women within the Department of Justice. Choices (A), (B), and (C) are all incorrect because those issues do not reflect the progressive agenda.

17. (B) NARAL Pro-Choice America is a special-interest organization that engages in political action and advocacy efforts to oppose restrictions on abortion and expand access to abortion. NARAL is often used as a short form of the name. Since Emily’s List supports pro-choice Democratic candidates, Emily’s List would certainly support the goals of NARAL. Choice (A) is incorrect because it is a conservative pro-business group while choices (C) and (D) are also incorrect because they are opposed to abortion rights, unlike Emily’s List.

18. (B) The map illustrates the various voter-identification requirements states have adopted and the type of identification required to vote. These states inform the voter where they can acquire a form of photo identification, usually a driver’s license or other government recognized document with a photo identification. The question that arises is whether the passage of these laws affects voter turnout and suppresses voting by Democratic-leaning groups. The Democratic Party has challenged many of these laws because it receives complaints from minorities, the elderly, people who cannot drive, and college students that cannot obtain a valid government-issued photo-identification document. Choice (A) is incorrect because even though that is the argument used by states that pass these laws, Democrats challenge them because they say there is very little evidence that there has been voter fraud. Choice (C) is incorrect because voter turnout usually decreases after the law goes into effect because not every voter can get voter-identification cards with photos. Choice (D) is incorrect because in many states that have voter-identification laws, college students from out of state often do not have state-recognized photo identification and therefore cannot vote.

19. (B) States that have passed voter-identification laws claim these laws are necessary to protect the integrity of the ballot. Attorneys general of these states assert it is easy to vote more than once, how voter rolls are inaccurate, and how important it is to provide a photo-identification document that can be validated before the person enters the voting booth. Choice (A) is incorrect because even with valid government-issued photo-identification documents, states still must issue a provisional ballot, a ballot that is used when the voter insists that the local Board of Election made a mistake in denying that person the right to vote. Choice (C) is incorrect because after photo-identification laws have been implemented, there has been a decrease in voter turnout. Choice (D) is incorrect because early voting is part of the voting process and, in the case of mail ballots, voter identification is not required.

20. (C) The political practice of gerrymandering, the drawing of legislative districts in a political manner that favors one political party or a candidate, started in 1812 when Governor Elbridge Gerry of Massachusetts signed into law a redistricting proposal approved by the state legislature that looked like a salamander. The Boston Gazette wrote an editorial giving the name Gerrymander to the shape of the district. Choice (C) is correct because the cartoon depicts the unfair drawing of the legislative districts. Choice (A) is incorrect because it did not call for the Supreme Court to rule on its constitutionality. Choice (B) is incorrect because Democrats and Republicans did not exist in 1812. Choice (D) is incorrect because the cartoon refers to the district “formed into a monster.” Thus, the editorial did not believe it was legitimate.

21. (A) Shaw v Reno (1993) is the Supreme Court case which raised the issue of whether majority-minority districts in North Carolina were legal. The decision described the shape of the new district as “bizarre” and said such a district “bears an uncomfortable resemblance to political apartheid.” Choice (B) is incorrect because Citizens United v Federal Election Commission (2010) is a Supreme Court case that dealt with campaign finance laws. Choice (C) is incorrect because Gideon v Wainwright (1963) was a Supreme Court case that resulted in the court establishing that the accused has the right to an attorney even if he or she cannot afford one. Choice (D) is incorrect because Marbury v Madison (1803) was the landmark case that established judicial review and resulted in declaring a congressional law unconstitutional. The question of the constitutionality of this district was not relevant.

22. (C) The Supreme Court case Citizens United v Federal Election Commission (2010) is a landmark U.S. constitutional law and corporate law case dealing with regulation of campaign spending by organizations. The court’s decision enabled organizations to spend an unlimited amount of soft money, money that is not regulated, because the court equated money spent for political advertising by these groups as a form of free speech protected by the First Amendment. Choice (A) is incorrect because public financing of presidential campaigns has strict limits set by law and does not allow for unlimited soft money. Candidates running for president have for the most part opted not to accept public financing and instead raise their own funds. Choice (B) is incorrect because only laws that increased hard money (regulated money) limits were passed. Choice (D) is incorrect because contributions to candidates by foreign countries is illegal.

23. (A) The chart illustrates how expensive congressional and presidential campaigns are, and how the cost of these elections has increased from 1998 to 2016. Candidates for Congress and the presidency have to raise enough money to support their campaigns: hire staff, pay for political advertisement, hire pollsters, and so on. The most effective way a candidate gets funds is by creating a political action committee. PACs act independently of the candidate, with their main purpose being to raise money for the candidate. Choices (B), (C), and (D) are all ways candidates can solicit funds. At a town meeting, the candidate can point out that the public can contribute on the candidate’s website. A candidate can run an ad soliciting funds, and political consultants can give advice on the best way to raise funds. But these methods are not as effective as creating a political action committee.

24. (B) The Voting Rights Act of 1965 was passed in response to procedures set up by mostly Southern states that impeded minority voter registration, minority representation in state and national legislatures, and made voting more difficult, all resulting in discrimination against African Americans. A key component of the law was to have the Justice Department monitor state voting where there had been instances of discrimination. Choice (A) is incorrect because the act also made literacy tests illegal. Choice (C) is incorrect because state legislatures were responsible for redrawing voting districts after the national census was taken. Choice (D) is incorrect because states still controlled voter registration after the Justice Department ordered any changes.

25. (D) Media-centered coverage of political campaigns has been dominated by the horse-race aspect of the campaign, rather than policy issues. With a 24-hour news cycle and social media outlets enabling candidates and the public to respond to any breaking news event, the coverage becomes the “story of the day” with constant discussion of how it affects the Election Day outcome. This story can be candidate-created, media-created, or created by an outside event. Choice (A) is incorrect because party loyalty is lessened in a media-centered campaign. Choice (B) is incorrect because a media-centered campaign also results in the coverage of negative campaigning by the candidates. Voter satisfaction is reduced because of that factor. Choice (C) is incorrect because there is greater voter reliance on social media and other forms of media for candidate information.

26. (B) Choice (B) is the correct answer because it illustrates that an implied power is one that is not specified in the Constitution. In this case, Congress passed the Civil Rights Act of 1964 using the Commerce Clause. Choices (A), (C), and (D) are all incorrect because they are delegated (also called enumerated) powers of Congress.

27. (D) When a potential candidate is deciding to run for president, he or she competes in what is called the invisible primary. The goal for the candidate is to get well known in Iowa, which holds the first caucus vote, and in the early primary states of New Hampshire, Nevada, and South Carolina. The candidate travels to those states to gauge public opinion, organize a grassroots ground game, and raise funds. Choice (A) is incorrect because the purchase of political ads in the candidate’s home state would be counterproductive, because the candidate is well known there. Choice (B) is incorrect because a running mate is chosen after the candidate is the presumptive nominee. Choice (C) is incorrect because a campaign slogan is a part of the candidate’s ground game in the early-voting states.

28. (D) The president sets an agenda of priorities. If one of them is the passing of a Clean Air and Clean Water Act, the president must win public and congressional support for the law. The State of the Union address can accomplish that. The president is able to speak directly to the American people and Congress about the importance of the legislation. Choice (A) is incorrect because there needs to be a national law that addresses the problem of carbon emissions. Since these emissions cross state lines, even if some states passed their own legislation, it would not solve the problem. Choice (B) is incorrect because the Environmental Protection Agency writes regulations after the law is passed. Choice (C) is incorrect because the Attorney General does not have the responsibility to investigate factories for abuses. The Department of Justice and the Environmental Protection Agency can bring a lawsuit against factories that violate the law.

29. (B) Gun-control legislation is one of the most difficult issues to get a consensus on from Congress and the National Rifle Association (NRA) even when over 90 percent of the public is in favor of laws that would require background checks at gun shows. The NRA has come out against such laws even though many of its members are in favor of it. If a senator is against the law, that senator would seek the help of the NRA to defeat the bill. The senator would also get other senators who are against the legislation to be part of a filibuster that would hold up a vote until 60 votes cut off debate. If there are not 60 votes, the legislation will not pass. Choice (A) is incorrect because senators do not lobby the Supreme Court. Choice (C) is incorrect because a cloture vote is the means by which the Senate attempts to end a filibuster. Choice (D) is incorrect because a discharge petition is a means by which a representative can get a piece of legislation on the floor of the House of Representatives.

30. (A) The Electoral College has played a pivotal role in two presidential elections in the twenty-first century when candidates winning the popular vote lost the election because their opponent won a majority of the Electoral College. A candidate must win 270 electoral votes to be elected president. If there are third-party candidates who split electoral votes, or if an elector does not vote for the candidate, it is possible that neither candidate would receive a majority of the electoral votes. It is also possible for there to be a tie in the Electoral College vote. In either case, when neither major party candidate receives a majority of the Electoral College votes, each state delegation in the House of Representatives votes, and the candidate getting a majority of the state votes becomes president. If there is a tie vote, the House keeps on voting until a candidate wins. If no candidate wins by inauguration day, the vice-president elect, who was chosen by a majority in the Senate, becomes acting president until the House selects the president. Choices (B), (C), and (D) are all incorrect answers and do not reflect what happens in the Electoral College.

31. (D) The First Amendment to the Constitution prohibits Congress from violating the right of free speech, guarantees freedom of the press and freedom of assembly, and gives people the right to exercise their religious beliefs while prohibiting Congress from establishing a state religion. Therefore, there is a balance between secular interests that would prevent such practices as prayer in school but would not prohibit a person wearing a skull cap or saying a prayer that is not organized by the school. Choice (A) is incorrect because there is no conflict between the free-speech provision and the separation of church and state. Choice (B) is incorrect because you are able to wear religious clothing such as a skull cap. Choice (C) is incorrect because the government can fund what are termed faith-based initiatives, where the government funds religious organizations to develop programs that help the poor.

32. (B) The principle of judicial activism is carried out by justices who believe the Constitution can be interpreted based on changing times. The Supreme Court’s decision on Brown v Board of Education (1954) overturned the precedent—established by Plessy v Ferguson (1896)—which established the concept that separate but equal does not constitute discrimination. In overturning Plessy, the court exercised judicial activism because it struck down an established precedent. Choice (A) is incorrect because constitutional originalism relies on what justices believe is the original intent of the Constitution. Choice (C) is incorrect because the principle of judicial restraint generally upholds precedent. Choice (D) is incorrect because the oversight of the judiciary is performed by an independent agency.

33. (C) Federal bureaucracies have the responsibility of organizing and running federal agencies as well as developing and implementing policy that is either passed by the Congress or initiated by the president. They accomplish these functions by coordinating their own resources and personnel to achieve policy goals. Choice (A) is incorrect because bureaucracies cannot abolish another agency. Only Congress can do that. Choice (B) is incorrect because the Congress determines the budgets of bureaucracies. Choice (D) is incorrect because Congress provides oversight of bureaucracies, not the other way around.

34. (C) The Fourth Amendment has two contrasting sections: the right of people to privacy and the right of the government to get information it needs as long as a legal search warrant is obtained based on the legal principle of probable cause. The Supreme Court has interpreted the privacy clause to include a woman’s right to decide whether she wants to have an abortion. Thus, a state passing a law that bans abortions before the twentieth week of pregnancy could violate the Roe v Wade decision. Choice (A) is incorrect because the police can obtain search warrants to get information from a cell phone. Choices (B) and (D) are incorrect because those situations do not apply to the Fourth Amendment.

35. (B) The required case Engel v Vitale (1962) established that a nondenominational prayer (a prayer that only has the word God and is not affiliated with a religion) that is said in a public school violates the establishment clause of the First Amendment. Subsequent to that decision, the court also ruled in Lee v Weisman (1992) that a school that invited a rabbi and priest to say a prayer at graduation also violated the principle set forth in Engel. Choice (A) is incorrect because the court allowed the funding of Bible groups. Choice (C) is incorrect because schools can teach a comparative religion course. Choice (D) is incorrect because the court also found that students do not have to attend school past the age of 16 in cases where it violated the religious views of the parents.

36. (B) The No Child Left Behind Act of 2001 and Every Student Succeeds Act of 2015 created Department of Education funding guidelines that set standards for states to adopt if they wanted to receive federal aid. These standards included increased testing in math and science and a new common core curriculum that many local school boards were against. The Tenth Amendment Reserved Power clause states that powers not given to the federal government in the Constitution are reserved to the states. Since education is not a stated power given to the federal government, states claim that the Department of Education is giving states an unfunded mandate unless the states adopt the new standards that were part of the law. Choice (A) is incorrect because the Privilege and Immunity clause of the Fourteenth Amendment protects the rights of citizens in states against abuses by the federal government. Choice (C) is incorrect because the Full Faith and Credit clause speaks to the credit of the federal government being protected, and to states having the right to make laws that are recognized in other states. Choice (D) is incorrect because the Supremacy clause states that federal laws that conflict with state laws will prevail.

37. (A) When a case is brought directly to the Supreme Court, the review is called original jurisdiction. This jurisdiction is defined as cases that deal with a conflict between states or that involve a lawsuit related to a foreign government. Thus, when New York and New Jersey had a conflict over boundaries related to Ellis Island, the case went directly to the Supreme Court because it was original jurisdiction. Choices (B) and (D) are incorrect because those cases are heard on appeal and, when the court accepts them, it is called appellate jurisdiction. Choice (C) is incorrect because even though President Trump’s executive order impacts immigration from certain foreign countries, the case does not fit the criteria of original jurisdiction and was heard on appeal from the lower courts.

38. (D) The Constitution has a provision that describes the confirmation process. A simple majority is necessary for presidential appointments. These include ambassadors, federal judges (including Supreme Court justices), and cabinet-level positions. The White House staff including the press secretary (choice (A)), the chief of staff (choice (B)), and the counsel (choice (C)) are all considered the president’s staff and are not subject to confirmation.

39. (D) The difference between a tracking poll and an exit poll is that a tracking poll is used before an election and measures which candidate is leading the race, while an exit poll is taken after a voter leaves the election booth on Election Day. Both polls use random sampling and are scientifically done. Choice (A) is incorrect because a 3.5 percent margin of error is within polling norms. Choice (B) is incorrect because tracking polls use scientific samples, while Choice (C) is incorrect because exit polls may or may not have a large sample.

40. (A) The iron-triangle concept relates to how policy gets passed when a law is proposed or money is requested for programs. The triangle represents the interrelationship between special-interest groups (and the lobbyists who represent them) trying to influence a congressional committee that drafts the legislation and the input from the bureaucratic agency that the policy or program impacts. Thus, a lobbyist would testify before a committee, lobby members of Congress, and get support from the Defense Department. Choices (B), (C), and (D) are incorrect because those answers represent a combination that does not reflect the iron-triangle concept.

41. (C) A “lame duck” politician refers to an officeholder in the last year of a term for which the politician cannot run again. The actual origin of the term has nothing to do with politics, however, and is quite specific in meaning. It comes from the London Stock Market and referred to investors who were unable to pay their debts. Because a president has limited political capital in the last year of holding office, the term lame duck applies. In President Obama’s last term, there was a vacancy on the Supreme Court after Associate Justice Antonin Scalia died. Even though the president nominated a federal Court of Appeals justice as a replacement, the Senate majority leader would not hold hearings, using the argument that the president is a lame duck and the next president should be able to appoint Scalia’s successor. Choice (A) is incorrect because Congress would pass emergency funding if a natural disaster occurs. Choice (B) is incorrect because the president can still issue an executive order during the last term. Choice (D) is incorrect because raising the debt ceiling is mandated by law.

42. (B) Equality of opportunity is defined as a philosophy of government that encourages providing a means for people to succeed by assuring everyone can have access to essential social programs. This philosophy differs from equality of outcome, where the government not only provides the means but also provides the monetary resources for the person to succeed. Choice (A) is incorrect because that is an example of individualism. Choices (C) and (D) are incorrect because those are examples of equality of outcome.

43. (C) The Anti-Federalist paper Brutus 1 represented the argument made by the Anti-Federalists, who opposed ratification of the United States Constitution. In Brutus 1, written by Robert Yates, Patrick Henry, and John DeWitt, the argument is set forth that a federal government under the Constitution would be despotic, and the only way to counter that would be an inclusion of the Bill of Rights. In Federalist No. 10, Alexander Hamilton talked about the danger of factions. He believed a consequence of factions, if not dealt with, would be the majority threatening minority rights. Choice (A) is incorrect because Brutus 1 did not favor the necessary and proper clause. Choice (B) is incorrect because Brutus 1 was against the Supremacy clause, and federalists supported it. Choice (D) is incorrect because Brutus 1 was against Congress having the right to tax.

44. (B) The required cases of Baker v Carr (1961) and Shaw v Reno (1993) both dealt with the issue of apportionment. Baker dealt with an imbalance of rural and urban districts and established the principle of “one man, one vote.” Shaw dealt with political gerrymandering of congressional districts and ruled that majority-minority districts were unconstitutional. Choice (A) is incorrect because the Shaw decision ruled that gerrymandered majority-minority districts were illegal. Choice (C) is incorrect because the Baker case ruled that how rural and urban districts were apportioned was illegal, and the Shaw case used the Voting Rights Act of 1965 in the decision. Choice (D) is incorrect because separation of powers was not an issue in the Baker case and race-based hiring was not part of the Shaw case.

45. (D) Delegated powers of the president are listed in Article II of the Constitution and include powers such as serving as commander in chief, the power to grant pardons, and the power to sign or veto laws. Enumerated or listed powers of Congress are in Article I Section 8 of the Constitution and include such powers as the power to declare war, the power to regulate interstate commerce, the power to make all laws necessary and proper, and the power to establish rules of naturalization (immigrants becoming citizens). Choice (A) is incorrect because the president does not have the power to declare war. Choice (B) is incorrect because the president has the power to appoint justices. Choice (C) is incorrect because there is no listed power that enables the president to issue executive orders—that is an inherent power the president has assumed as a result of being chief executive.

46. (A) Judicial activism is a philosophy characterized by Supreme Court justices making decisions that overturn precedent, also known as stare decisis, where the four justices accept a high number of cases for oral argument, also known as certiorari, and when the justices believe that the Constitution is a document that can be interpreted based on changing conditions. Judicial restraint is a philosophy characterized by decisions that uphold precedent, where the four justices accept a limited number of cases for oral argument, usually around 75 in a term, and believe in a strict interpretation of the Constitution. Choice (B) is incorrect because the answer reflects the opposite philosophy. Choice (C) is incorrect because accepting an amicus brief, also known as a friend of the court brief, is standard procedure for the court. Choice (D) is incorrect because the Supreme Court does not allow live coverage of oral arguments.

47. (A) The Republican Party and Democratic Party have differing views regarding the environment—views that are reflected in their respective party platforms and votes in Congress. Democrats believe the federal government should pass laws regulating carbon emissions, Choice (A), and Republicans believe the federal government should allow and promote “clean” coal production. Choice (B) is incorrect because Republicans believe we should abandon climate-control treaties while the Democrats believe the United States should be part of climate-change treaties. Choice (C) is incorrect because Democrats believe the Environmental Protection Agency should set strict standards on clean air and clean water, while the Republicans believe there should be fewer regulations. Choice (D) is incorrect because Democrats do not believe National Parks should be open to private investment.

48. (D) Choice (D) is the correct answer because unlike a primary a caucus has a different method of selecting delegates. The Iowa caucus is a good example of this. Before votes are cast, each candidate has representatives making a case for them. Choice (A) is incorrect because the Iowa caucus is only open to people registered in a political party. Choice (B) is incorrect because more states use a primary system. Choice (C) is incorrect because, by the decisions of both political party national committees and custom, the Iowa caucus has been the first presidential caucus since 1972.

49. (B) The Voting Rights Act of 1965 was passed with the primary purpose of dealing with states that had a pattern of discrimination in granting the right to vote. It primarily impacted states in the South, and federal marshals were sent to monitor and make sure that African Americans and other minorities could register to vote. Choice (A) is factually incorrect. Choice (C) is incorrect because state legislatures redraw voting districts. Choice (D) is incorrect because the federal government did not take control; rather, they monitored voting registration.

50. (C) Choice (C) is correct because if there is divided government where one branch is controlled by one party and another branch is controlled by the other, compromise is one of the ways that often breaks gridlock. This occurred after the Republicans took control of Congress in 1994. Choice (A) is incorrect because the president’s power is weakened when there is divided government. Choice (B) is incorrect because that is not a consequence of divided government. Choice (D) is not correct because the president still has veto power if Congress is controlled by the other party.

51. (C) “Earmarks” are defined as “add-ons” to proposed bills—extra benefits that individual representatives ask for in return for their vote for a bill. Most of the time earmarks give the voting representative’s home district some financial boon in the form of funds or programs. Choice (A) is incorrect because earmarks add more money to proposed legislation. Choice (B) is incorrect because cutting off debate is done through a vote. Choice (D) is incorrect because bills are sent back to committee if they do not have enough votes.

52. (A) The Fifth Amendment creates a number of due-process rights relevant to both criminal and civil legal proceedings. In criminal cases, the Fifth Amendment guarantees the right to a grand jury, forbids “double jeopardy,” and protects against self-incrimination. The Miranda rights describe the due-process right of remaining silent, which is the same as protection against self-incrimination. Choice (B) is incorrect because the Fourth Amendment prohibits police from obtaining evidence without a search warrant. Choices (C) and (D) are incorrect because the right to an attorney and the right to confront a witness are Sixth Amendment protections.

53. (A) Devolution is a federalism principle that transfers powers from the federal government to the states, such as control over education and over unfunded mandates that states are forced to spend money for. Devolution echoes the intent of the Reserved Power clause of the Tenth Amendment. Choice (B) is incorrect because the Full Faith and Credit clause speaks to a guarantee backing up United States credit and the recognition by one state of laws that have been passed by another state. Choice (C) is incorrect because the Equal Protection Clause of the Fourteenth Amendment deals with protecting the rights of individuals from state and federal abuses. Choice (D) is incorrect because eminent domain is the right the government has to take private property for the public good, as long as the government pays for the value of the property.

54. (B) Choice (B) is correct because, by definition, social media outlets include Facebook, Twitter, and bloggers. The people using these outlets may or may not be journalists who are trained to report the news. Therefore, unsubstantiated information may be presented, creating a belief that social media sites may not be reliable, even though they are so heavily used. Choice (A) is incorrect because there is less reliance on the mainstream media because so many other media outlets are reporting news. Choice (C) is incorrect because, even though there is a greater concentration of news media ownership, this is not a result of the increase of social media outlets. Choice (D) is incorrect because Internet postings are largely unregulated.

55. (A) Part of the definition of congressional oversight is a congressional committee reviewing the budget and actions of bureaucratic agencies. Congressional oversight can also include reviewing actions taken by the executive branch, including the president. Witnesses can be called to testify, and committee members can ask questions. After the committee completes its oversight, Congress may take action based on the findings of the committee. Choice (B) is incorrect because drafting appropriations is part of the budgetary process. Choice (C) is incorrect because setting time limits for debate is part of the legislative process. Choice (D) is incorrect because the impeachment of a president is based on charges brought to the Judiciary Committee.

Section 2: Free–Response Questions

Explanation and Discussion of Question 1: Concept Application Scenario Question

This question is based on a press release from the United States Census Bureau to tell the public that the census has fulfilled its legal obligation of informing Congress about planned changes for the 2020 Census.

Part A asks you to describe from where the Census Bureau gets its authority to address the issues raised in the passage. The first Census was completed in 1790 as a result of the requirement found in the United States Constitution in Article II Section 1, directing the government to take a census of the population. That was done every ten years, and subsequent legislation passed by Congress gave specific directions to the Census Bureau. Today, the controlling law for the U.S. Census required the population to be counted on or about April 1, 1980, then every ten years after that. In the intervening years between census-taking the Census Bureau is required to gather statistics about the residents of the United States for use by Congress. Unlike the ten-year censuses, the count done in the intervening years need not be actual headcounts, but may use statistical sampling methods to get a reasonable approximation of the population.

Part B asks you to use the information provided in Part A and explain how the Census Bureau is planning on executing its authority and a description of one problem that could arise as a result of how the 2020 Census will be taken. For the first time, an online response option will be part of the 2020 Census. The Census Bureau hopes this method will increase accuracy, improve design of questions and the quality of data. The bureau also hopes that with an online option, more people will be able to access forms rather than only getting them in the mail. A problem that could arise from this process is whether the online response will give the Census Bureau complete and accurate information. This is further complicated by the fact that it could be harder to reach minorities and undocumented immigrants who do not get mailed forms and do not have access to online materials.

Part C asks you to evaluate the possible consequences the 2020 Census will have on the political institutions of state and federal governments. Once the census is completed, the returns must be made available within nine months in order to reapportion the number of seats in the House of Representatives to each of the states. If a state increases in population it may also gain Electoral College votes in the next presidential election, and if a state loses population, it may lose Electoral College votes. Census results are also used by state legislatures to apportion their state’s legislative districts. An increase or decrease in population in a district may result in the redrawing of that district, which could have political consequences such as one party gaining seats based on the configuration of the new districts.

Scoring Rubric

This question is 3 points.

Part A

Scoring Criteria

Applicable Units

Scoring Guidelines and Examples of Correct Answers

Describe from where the Census Bureau gets it authority.

Foundations of American Democracy (The Constitution), Interactions Among the Branches of Government (Baker v Carr, Shaw v Reno), Political Participation

Examples must include the Article in the Constitution that states that Congress directs the Census Bureau to conduct a census, and laws are passed that update that requirement. (1 pt.)

Part B

Scoring Criteria

Applicable Units

Scoring Guidelines and Examples of Correct Answers

Explain how the Census Bureau plans on executing the constitutional authority and congressional authority and one problem that could result as a result of the method the Census Bureau is using.

Interactions Among the Branches of Government (apportionment and gerrymandering)

The answer must explain what methodology the Census Bureau is going to use. Correct answers include mailed forms and online forms. Problems that could arise include accuracy, fairness of questions, and whether the online method will improve the census statistics. (1 pt.)

Part C

Scoring Criteria

Applicable Units

Scoring Guidelines and Examples of Correct Answers

Explain what consequences the 2020 Census will have on the political institutions of state and federal governments.

Political Participation (voting)

Correct answers must explain both that the results of the Census will be given to the states, which must then reapportion both congressional and legislative districts, and also the impact this will have on presidential, congressional, and state legislative elections. (1 pt.)

Explanation and Discussion of Question 2: Supreme Court Comparison Question

This question is based on an understanding of one of the required Supreme Court cases in the newly redesigned curriculum: Roe v Wade (1973). The issue of privacy rights guaranteed in the Fourth Amendment and due process rights guaranteed by the Fourteenth Amendment applies to both the Roe v Wade case and the Planned Parenthood of Southeastern Pennsylvania v Casey (1992) case. The text in the question summarizes the facts and issues raised by the Casey case.

Part A asks you to identify the constitutional clause that both Roe v Wade and Casey have in common. The Fourth Amendment to the United States Constitution guarantees a person to be secure in his or her person, a foundational privacy right. The court in Roe v Wade extended that privacy right to first and second trimester abortions and, to a more limited extent, to the third trimester. In subsequent cases, like Casey, the Supreme Court had to determine whether the precedent established in Roe v Wade should be upheld, and whether the state had the right under the Roe standards to impose restrictions—even though those restrictions are not allowed to be an undue burden on a woman getting an abortion.

Part B directs you to show that even though the constitutional issue in Part A was used in both cases, why the decision of Roe v Wade was different than the one reached in Casey. The Supreme Court ruled that even though the precedent established in the Roe case, that women can have abortions in the first and second trimesters and, to a more limited extent, in the third, Pennsylvania had the right to require that a woman get “informed consent” prior to the procedure, and after a doctor explains that abortions could be detrimental to the health of the mother. The state also had the right to impose on an underage woman the requirement that she get parental consent or seek judicial bypass. The court ruled against the state in the part of the law that required spousal notification and let stand a provision that had an emergency definition that provided for an abortion if the pregnancy threatened the life of the mother or created a serious health risk.

Part C wants you to describe what happened as a result of the Supreme Court decision. Since the court ruled that the state had the right to impose restrictions on a woman seeking an abortion, even in the first and second trimesters, as long as the restrictions did not create an undue burden on the woman, it made it more difficult for women to get an abortion in Pennsylvania. These restrictions were limited to Pennsylvania but, as a result of the court ruling, other states passed similar laws. Special-interest groups such as Planned Parenthood were pleased that the precedent in the Roe case was upheld and that spousal notification was struck down, but they were critical of the restrictions imposed on women who wanted abortions.

Scoring Rubric

This question is 4 points.

Part A

Scoring Criteria

Applicable Units

Scoring Guidelines and Examples of Correct Answers

Identify the constitutional issues the majority opinion uses in both cases.

Foundations of American Democracy (Judicial Review), Civil Rights and Civil Liberties (Civil Liberties, the Fourth Amendment), American Political Beliefs and Behavior (government involvement in social issues)

Explanation of what the Fourth Amendment privacy provision and the Fourteenth Amendment’s due process clause means and how they apply to the Roe v Wade and Casey cases. (1 pt.)

Part B

Scoring Criteria

Applicable Units

Scoring Guidelines and Examples of Correct Answers

Explain why the decision reached in Planned Parenthood v Casey was different than the one reached in Roe, even though they were both Fourth Amendment cases.

Foundations of American Democracy (Judicial Review), Civil Rights and Civil Liberties (Civil Liberties, the Fourth Amendment), American Political Beliefs and Behavior (government involvement in social issues)

Even though the Fourth Amendment right of privacy guarantee was argued in both the Casey and Roe cases, because the court ruled in Casey that state restrictions on abortion in the form of consent was legal, the decision limited the scope of Roe v Wade to establish a new precedent. Correct answers include any of the restrictions that Casey allowed as well as an explanation of how it differed from Roe. (2 pts.)

Part C

Scoring Criteria

Applicable Units

Scoring Guidelines and Examples of Correct Answers

Describe what happened as a result of the decision made by the Supreme Court in the Planned Parenthood v Casey case and how a special-interest group reacted to the decision.

Foundations of American Democracy (Judicial Review), Civil Rights and Civil Liberties (Civil Liberties, the Fourth Amendment), American Political Beliefs and Behavior (government involvement in social issues)

Because the court was split in the Planned Parenthood v Casey decision, the part of the decision that upheld Roe’s precedent continued to keep abortion legal as reflected in the Roe decision. However, since the court upheld the right of Pennsylvania to establish new restrictions on women seeking an abortion, the results made it more difficult for women to get abortions if they had to get consent, and made it legal for other states to pass similar laws. The special-interest group Planned Parenthood favored the part of the decision that upheld Roe v Wade but was critical of the restrictions Pennsylvania imposed. Another correct answer would be to include any right to life special-interest group that would have been critical of the court upholding the Roe v Wade precedent. They would support the restrictions. (1 pt.)

Explanation and Discussion Question 3: Free-Response Stimulus

This question presents a graph of the United States Federal Government Income and Outlay (expenses) measured in trillions of dollars, prepared by the Office of Management and Budget (OMB), whose job is to prepare statistics and recommendations related to the federal budget process.

Part A asks you to identify a common feature of the federal budget between 1982 and 2012. The trend of the federal budget between 1982 and 1996 and between 2002 and 2012 was that the budget had a deficit that increased between 1982 and 1996 and again between 2002 and 2012.

Part B asks you for the identification of a deviation that took place in the federal budget between 1998 and 2000. The only period that the federal budget did not have a deficit was between 1998 and 2000 when there was a surplus, and the federal government took in more money from taxes than it spent on government programs. This occurred because Congress passed a budget that was signed into law that provided for a surplus rather than using the extra money to bring down the national debt or to increase the spending of discretionary programs. As a result of the surplus, there was a debate regarding what to do with it. The surplus was short-lived as the budget again went into deficit spending after 9/11 and our involvement in the wars against Iraq and Afghanistan.

Part C asks you to explain how the budget illustrates the fiscal policy of the United States Congress, which passes the yearly budget bills, and the presidents who sign the budget into law. Fiscal policy is defined as how the government attempts to manage the economy through taxation, spending, and borrowing. Democrats and Republicans have different philosophies concerning budget priorities. Republicans traditionally favor lower taxes, smaller deficits, and spending priorities that include defense spending and spending that would encourage job growth. Democrats favor spending programs that protect the safety-net programs of Social Security, Medicare, and Medicaid. Democrats also favor a tax policy that is most beneficial to the middle class rather than to the wealthy. It is the clash of these philosophies that results in the budget trends found in the graph. During the short period when there was a budget surplus, both the Republican Congress and the Democratic president were able to agree on approving that budget priority.

Scoring Rubric

This question is worth 4 points.

Part A

Scoring Criteria

Applicable Units

Scoring Guidelines and Examples of Correct Answers

Identify a common feature of the federal budget between 1982 and 2012.

Foundations of American Democracy (U.S. Constitution—Appropriation Power of Congress); Interactions Among the Branches of Government (Laws passed by Congress related to budget process); Political Participation (Competing interests among special-interest groups related to the budget)

The correct answer includes the two periods (1982–1998 and 2002–2012) that had budget deficits where there were more outlays than income. (1 pt.)

Part B

Scoring Criteria

Applicable Units

Scoring Guidelines and Examples of Correct Answers

The identification of a budget deviation that took place in the federal budget between 1998 and 2000.

Foundations of American Democracy (U.S. Constitution—Appropriation Power of Congress); Interactions Among the Branches of Government (Laws passed by Congress related to budget process); Political Participation (Competing interests among special-interest groups related to the budget)

Accurately describe that the only period in which the federal budget did not have a deficit was between 1998 and 2000, when there was a surplus and the federal government took in more money from taxes than it spent on government programs. (1 pt.)

Part C

Scoring Criteria

Applicable Units

Scoring Guidelines and Examples of Correct Answers

Explain how the budget illustrates the fiscal policy of Congress and the president from 2002 to 2012.

Foundations of American Democracy (U.S. Constitution—Appropriation Power of Congress); Interactions Among the Branches of Government (Laws passed by Congress related to budget process); Political Participation (Competing interests among special-interest groups related to the budget)

Gives an explanation of the meaning of fiscal policy and how it impacts the federal budget. Answer should include references to taxing, spending, and examples of the types of programs that comprise the federal budget. Two points for a correct answer with an example to support the answer. (2 pts.)

Free-Response Question 4: Argument Question (6 pts.)

This question asks the student to develop an argument or thesis that explains which of the ideas that contributed to limited government—including natural rights, popular sovereignty, and the social contract—best accomplishes the goals set forth in the Declaration of Independence and the U.S. Constitution.

The question also asks for one piece of accurate and relevant information from one required document and one piece of accurate and relevant information from a second required document or any relevant document you studied that relates to the question. Federalist No. 10 was written by Hamilton to explain the problems that would occur if factions, which Hamilton believed were inevitable, were not kept in check. The Declaration of Independence describes unalienable rights as the cornerstone of natural rights. As a consequence of these rights, limited governments receive their power from the consent of the governed in the form of popular sovereignty. The Constitution is the blueprint of how limited government works with separation of powers and checks and balances. Other foundational documents that can be used include Federalist No. 51 to lay out the reasons for three independent branches of government. This would ensure the people would have their liberty. Another document that was optional in the curriculum was Locke’s Second Treatise of Government (1690), in which he develops a number of notable themes. It begins with a depiction of the state of nature, wherein individuals are under no obligation to obey one another but are each themselves judge of what the law of nature requires.

There are four arguments you can use to answer this question. The first argues that natural rights comprise the most important factor that contributes to limited government. The second is that popular sovereignty is the most important contributing factor. The third argument is that the social contract is the most important factor. The last argument, and the argument that I would make, is that they are all equally important and address different aspects of limited government. Regardless of the argument you use, you must define limited government as the doctrines of natural rights and popular sovereignty, and the social contract. Limited Government was adopted by Jefferson and restricts the power of government, especially in the area of protecting the rights of the people. Popular sovereignty, is a political doctrine that advocates a government created by and subject to the will of the people. And the social contract that individuals have consented to, either explicitly or tacitly, is to surrender some of their freedoms and submit to the authority of the ruler (or to the decision of a majority), in exchange for protection of their remaining rights.

The last section asks you to respond to the opposing arguments. Therefore, if you argued that one of the philosophical ideas of limited government was its most important contributing factor, you would have to explain how the other two ideas had less of an impact on the success of limited government. If you chose the argument that they were all equally important, you would respond to each of the other factors and make the argument that they all had to work in tandem for a limited government to work.

Scoring Rubric

This question is worth 6 points.

Part A

Scoring Criteria

Applicable Units

Scoring Guidelines and Examples of Correct Answers

Develop an argument by choosing a position or thesis.

Foundations of American Democracy (Federalist No. 10, The Declaration of Independence, The Constitution, Federalist No. 51), American Ideologies and Beliefs (core beliefs)

The strongest thesis is that natural rights, popular sovereignty, and the social contract are all equally important factors that contribute to an effective limited government. Other thesis statements would take one of those factors and state that it was the key factor that made limited government work. Regardless of which argument you pick, you must be able to define limited government, natural rights, popular sovereignty, and the social contract and how they impact the effectiveness of limited government. (2 pts.)

Part B

Scoring Criteria

Applicable Units

Scoring Guidelines and Examples of Correct Answers

Support your position with TWO pieces of accurate and relevant information.

Foundations of American Democracy (The Constitution. Federalist No. 51), Interaction Among the Branches of Government (Judicial Branch exercising its power of Judicial Review) Civil Liberties and Civil Rights (Optional Supreme Court cases that support judicial activism and judicial restraint)

Federalist No. 10 was written by Hamilton to explain the problems that would occur if the inevitable factions that would arise were not kept in check. The Declaration of Independence describes unalienable rights as the cornerstone of natural rights. The court case Marbury v Madison (1803) established the principle of judicial review, which is the basis of either philosophy. Another foundational document you can use could be Federalist No. 51, which talks about checks and balances. The U.S. Constitution is the blueprint for how limited government works, with separation of powers and checks and balances. Another document that was optional in the curriculum was Locke’s Second Treatise of Government (1690), in which he develops a number of notable themes. It begins with a depiction of the state of nature, wherein individuals are under no obligation to obey one another but are each themselves judge of what the law of nature requires. (2 pts.)

Part C

Scoring Criteria

Applicable Units

Scoring Guidelines and Examples of Correct Answers

Summarize your argument by offering reasons why your proof supports your position and respond to the opposing point of view by refuting, conceding, or rebutting that point of view.

Foundations of American Democracy (Federalist No. 10, The Declaration of Independence, the Constitution. Federalist No. 51), American Ideologies and Beliefs (core beliefs)

Depending on the position you took, you would use the arguments made by the other philosophies to conclude that your position is the strongest one. For example, if you chose the argument that all three philosophies were equally as important to make limited government work, you would have to acknowledge the importance of the social contract, natural rights, and popular sovereignty. By doing so, it would make your position stronger, because your position reflects the strengths of all three philosophies. (2 pts.)

Since every test will be scored utilizing specific criteria for each question, there will be a different weighted score applied to every test. Please note that the multiple choice section is worth 50% of the test and the free response section is also worth 50% of the exam with each free response question worth 12.5%.

In the 2019 test students achieving a minimum multiple-choice score of 37.4 out of 55 and a total free response score of 9.1 out of 17 points received a 3. Students achieving a minimum multiple-choice score of 47.1 out of 55 points and a free response score of 13.6 out of 17 points received a 5.